PT3.S2.Q3 - In Europe, schoolchildren devote time during

btownsqueebtownsquee Alum Member
edited June 2018 in Logical Reasoning 1207 karma

I think this is a necessary assumption question. Correct me if I'm wrong please!

Summary of stimulus:
Context:
In the EU, schoolchildren do daily calisthenics during school.
In North America, schoolchildren rarely do daily calisthenics during school.

Premise:
Tests show that North American children are weaker, slower, and shorter-winded than European children.

Conclusion:
North American schoolchildren can be made physically fit -> North American schoolchildren do daily calisthenics

Notes:
1. lots of sets/subsets here: Children which contains schoolchildren which contains both schoolchildren in North America and schoolchildren in Europe. ALSO calisthenics which contains school calisthenics.
2. Is the argument assuming that the daily calisthenics are the reason that the EU schoolchildren are stronger than the North American children? Is he saying daily calisthenics is the only cause of EU schoolchildren being stronger than North American children? If so, then the author is using analogous reasoning by assuming EU schoolchildren are like North American schoolchildren to conclude that North American schoolchildren can be made physically fit if they do daily calisthenics.

Reading the ACs:
A) Children -> can be made physically fit by daily calisthenics. This should be crossed out since we don't need ALL children to be made physically fit when our conclusion is just about North American children. Is there another reason to eliminate A?
B ) Children -> can be made equally physically fit by daily calisthenics; I don't know if the children have to be made equally physically fit. They just need to get to a level where they are physically fit. Wrong.
C) We are concerned about physical fitness in this argument, not about health. Wrong.
D) This says school calisthenics are a necessary factor in EU children's physical fitness--this is what the author assumes.
E) We don't care about the children eating more nutritious diets/exercising daily--we care about calisthenics! Wrong.

Am I writing the conditionals in A and B wrong? I originally picked A because I thought it would guarantee the conclusion--which is wrong to do bc this isn't an SA question. Is A wrong for additional reasons that I've listed above?

Admin note: edited title

Comments

  • RunningOnEmptyRunningOnEmpty Alum Member
    40 karma

    Okay! This is my first time answering someone's question online. If there is anything wrong then I hope someone will point it out for me. Thanks in advance!

    Yes, this is a necessary assumption question.

    I think the key to the answer is the author's 'only if' conditional in the conclusion. If daily calisthenics is the only way to achieve physical fitness then:

    physically fit ---> daily calisthenics

    Basically the author's assumption is that there is no other way for a child to achieve the goal of becoming physically fit other than by doing daily calisthenics (answer choice D). If you fail that necessary assumption (that daily calisthenics is not the only way for children to achieve physical fitness) then the argument fails.

    Whereas in (A) it is saying:

    child --> could be fit by daily calisthenics

    Answer choice (A) is saying that if a child does daily calisthenics then they can get fit. According to this answer choice, it's a possibility that they become physically fit, it's not a necessity. So who does get fit and who doesn't? According to the author conditional statement in the conclusion, those who [successfully] do daily calisthenics (affirming the necessary condition).

    In the stimulus, the author does talk about how successful children are at calisthenics (which is what answer choice A is talking about). For the argument to be valid, the author does not have to assume that every child who does daily calisthenics is going to be successful, just that those who are successful will have achieved physical fitness.

    How do we know? Well, let's fail the assumption that every child who does daily calisthenics has to be successful. Does that result in making the argument fail? No, all it does is tells us that those who failed wouldn't have achieved physical fitness, but those who were successful would have achieved physical fitness.

    The author's assumption is not that anyone who tries to do daily calisthenics will be able to do it and as a result achieve physical fitness. Rather, the assumption is that if the children achieved physical fitness then then they did so by successfully doing daily calisthenics (affirming the necessary) which, in the author's opinion, is the only way to attain physical fitness:

    physically fit ---> daily calisthenics

    I hope this makes sense :neutral:

  • paulmv.benthempaulmv.benthem Alum Member
    1032 karma

    @btownsquee, my that is kinda a weird question. Way to go taking a crack it, @btownsquee & @maysa_t !

    @btownsquee said:
    I think this is a necessary assumption question. Correct me if I'm wrong please!

    Summary of stimulus:
    Context:
    In the EU, schoolchildren do daily calisthenics during school.
    In North America, schoolchildren rarely do daily calisthenics during school.

    Premise:
    Tests show that North American children are weaker, slower, and shorter-winded than European children.

    Conclusion:
    North American schoolchildren can be made physically fit -> North American schoolchildren do daily calisthenics

    To begin, let me say that I really dislike this passage. I think the conclusion is poorly worded as it pertains to the rest of the argument. That is, the argument makes reference to a relational degree of fitness between the two groups of children in the beginning, but then in couches its conclusion grammatically in such a way that it implicitly references some objective standard of physical fitness. To get the conclusion they are pushing, in addition to assuming the correct answer choice, we also have to assume that European children doing calisthenics are, actually, physically fit. To illustrate the problem, Joe weighs 900lbs and can only run 6 meters before passing out; though, this is 2 meters father than Mike can run...so, is Joe physically fit? I'm sure you can see the problem. The writers are wanting us to assume that a person can be understood to be physically fit if there is another person of group of persons who are less fit. But, as can be seen from the answer choices, that is not the assumption you're suppose to look for. (If this were a flaw question, this is the flaw I would anticipate.)

    At the same time, the question stem does NOT say that the correct answer choice is an assumption sufficient for validity, so maybe I'm just shadowboxing at this point. :P

    Anyway, now that I've gotten that off my chest...I would agree with much of what @maysa_t said above, "A" can be eliminated because it is a sufficiency-necessity reversal. I think the reason you list for eliminating "A" is a worthwhile one, but because the LSAT writers often employ answer choice with an necessity-sufficiency reversal, I think it's worthwhile to identify that in your analysis to that you're primed for when it rears its head again. The conclusion is claiming that calisthenics is necessary for physical fitness, but answer choice "A" inverses this relationship.

  • kelseyperkins1192kelseyperkins1192 Core Member
    66 karma

    Can someone write this question out in conditional lawgic for me? I'm so confused.

Sign In or Register to comment.